LSAT and Law School Admissions Forum

Get expert LSAT preparation and law school admissions advice from PowerScore Test Preparation.

User avatar
 Dave Killoran
PowerScore Staff
  • PowerScore Staff
  • Posts: 5853
  • Joined: Mar 25, 2011
|
#46112
Complete Question Explanation
(The complete setup for this game can be found here: lsat/viewtopic.php?t=17052)

The correct answer choice is (B)

The conditions in the question stem eliminate template #2 from consideration. Since there are still three templates in consideration and thus a considerable number of possibilities, it is best to consider each answer choice against all remaining templates.

For answer choice (A) only templates #3 and #4 have P toured on Monday; but neither have O on Thursday, and so answer choice (A) is incorrect.

For answer choice (B) only templates #3 and #4 could have P toured on Tuesday, and template #3 allows S to be toured on Wednesday, so answer choice (B) is correct.

For answer choice (C) only templates #3 and #4 have O toured on Tuesday. Template #4 does not have P toured on Friday, and so it does not apply; template #3 could have P toured on Friday, but to do so would violate the condition in the question stem requiring one other division beside S to be toured twice. Consequently, answer choice (C) is incorrect.

For answer choice (D), only template #1 applies, and if O were toured on Friday again, the condition in the question stem requiring one other division beside S to be toured twice would be violated. So answer choice (D) is incorrect.

Finally, for answer choice (E) only template #3 applies, but if P were toured on Friday, the condition in the question stem requiring one other division beside S to be toured twice would be violated. So answer choices (C), (D), and (E) each can be eliminated by the condition in the questions stem.

In this question it is easier to find the correct answer than it is to eliminate the incorrect answers.
 cardinal2017
  • Posts: 19
  • Joined: Oct 23, 2016
|
#35229
Hi, I chose (A) for this question, which is incorrect and I still can't figure out why it's wrong since the case in (A) actually works along with the original condition and the new condition placed by this question.

Can anybody help me please?

It's urgent!
 AthenaDalton
PowerScore Staff
  • PowerScore Staff
  • Posts: 296
  • Joined: May 02, 2017
|
#35271
Hi Cardinal,

Thanks for your question!

Answer choice (A) is incorrect because it forces a violation of the rule that if O is toured on Thursday, P must be toured on Friday.

Answer choice (A) tells us that P is toured on Monday and O is toured on Thursday:

M: P
T: __
W: __
Th: O
F: __

We know that when O is toured on Thursday, P is toured on Friday, giving us a schedule that looks like this:

M: P
T: __
W: __
Th: O
F: P

At this point, there would be room for [SS] to be toured on Tuesday and Wednesday, but there would be no other room for consecutive-day tours of either P or O.

Hope this helps!

Athena Dalton
 hope
  • Posts: 84
  • Joined: Feb 13, 2018
|
#44265
Help! On page 548 of the above listed book, I do not understand how B is the correct answer for number 15. Is C incorrect because it fails to place P in a consecutive position? And is the same issue of "consecutive" applicable to Answer E? Please let me know. Thank you.
 Adam Tyson
PowerScore Staff
  • PowerScore Staff
  • Posts: 5153
  • Joined: Apr 14, 2011
|
#44286
Hey there, Hope, let me see if I can help with that!

Your approach to this question should be to first eliminate any loser answers, ones that you can see right away are impossible. For example, you might have tried some "what if" scenarios while doing your original diagram using the conditional rule about O on Thursday and come up with these two possibilities when O is on Thursday:

PSSOP
SSOOP

You should be able to eliminate answer A quickly, because when O is Thursday and P is Monday, there is no division other than S that is toured on two consecutive days. After that, you may have to do a little testing to see what happens with your remaining answer choices.

Testing answer B, we start by doing what the answer tells us to do: put P on Tuesday and S on Wednesday:

_PS__

Now, S has to be part of an SS block, per the original rules, so we get this:

_PSS_

The rules also tell us that O cannot be Monday, and S only goes those two times, so that means Monday has to be P:

PPSS_

We have to get an O in there somewhere, and Friday is all that's left, so there we have it!:

PPSSO

This solution works, complies with all the original rules, and complies with the local restriction in the question (another consecutive pair besides the SS block, which in this case is the PP block on Monday and Tuesday). We have a winner!

Now, let's try answer C, starting by doing what the answer tells us to do, which is place O on Tuesday and P on Friday:

_O__P

There's only room for the SS block at Wednesday/Thursday, so let's put it there:

_OSSP

Now, to comply with the question's local restriction, we need another block of two divisions consecutively. The only way to do that is to put an O on Monday. But the second rule prohibits that! We cannot comply with all the rules, including the local restriction posed by the question and the requirements of answer C, and get a workable solution. That's why it's not the credited response to this question. It's not that there are not two consecutive Ps (although there are not), but that there is no consecutive pair other than the SS block.

Now let's try the same thing with answer E, since you asked! The answer says to put S on Wednesday and P on Friday, like so:

__S_P

We cannot put an O on Monday, but we have to place one somewhere. What if we put it on Tuesday? That would force the SS block to be on Wednesday and Thursday:

_OSSP

Nope, no way to get a second block of two consecutive divisions. Okay, let's try that O on Thursday instead, with the SS block on Tuesday/Wednesday:

_SSOP

We still can't get another block of two consecutive days of the same division! E won't work no matter which way you try to solve it. It''s out!

On these local questions, be sure to comply with the local restriction posed by the question, AND to remain compliant with all the original rules (unless instructed otherwise, as in a "rule substitution" question). When in doubt, draw it out!

I hope that helps!

Get the most out of your LSAT Prep Plus subscription.

Analyze and track your performance with our Testing and Analytics Package.